LSAT and Law School Admissions Forum

Get expert LSAT preparation and law school admissions advice from PowerScore Test Preparation.

User avatar
 KelseyWoods
PowerScore Staff
  • PowerScore Staff
  • Posts: 1079
  • Joined: Jun 26, 2013
|
#80549
This game is also discussed in our Podcast: LSAT Podcast Episode 70: The May 2020 LSAT-Flex Logic Games Section

Complete Question Explanation
(The complete setup for this game can be found here: viewtopic.php?t=33052)

The correct answer choice is (B).

This marks the beginning of a run of Local questions that require more time, which puts pressure on you since most students arrive at this point at the very end of the section.

For this Local question, you need to do a mini-diagram with O on the 3rd day. If O is on the 3rd day, we know that M cannot be on day 2 (or day 7) or day 4. Since we already know that H cannot be on day 1 or day 5, this really restricts where that HM block can now go: HM can go 4/5 or 7/8. I would do 2 diagrams for this--one to show HM in 4/5 and the other with HM in 7/8.

If HM are in 4/5, then M would also have to be 1st. J would have to then be 2nd (so it is before H) which means it would also have to be 7th. That just leaves slots 6 and 8 for L and G. They are randoms so either one of them could go in either slot.

If HM are in 7/8, then H would also have to be 2nd. J would then have to be 1st (so it is before both Hs) which means it would also have to be 5th. That just leaves slots 4 and 6 for L and G. They are randoms so either one of them could go in either slot.

Those are the only possibilities if O is 3rd. Thus, answer choice (B) could be true--H can be 2nd if HM are in 7/8. None of the other answer choices are possible.
Screen Shot 2021-01-04 at 3.09.49 PM.png

Answer choice (A): As shown in the diagram, only H or J can be performed on day 2, and thus this answer choice is incorrect.

Answer choice (B): This is the correct answer choice, as explained above.

Answer choice (C): As shown in the diagram, only J or M can be performed on day 1, and thus this answer choice is incorrect.

Answer choice (D): As shown in the diagram, only H or J can be performed on day 7, and thus this answer choice is incorrect.

Answer choice (E): As shown in the diagram, only H or J can be performed on day 7, and thus this answer choice is incorrect.
 kassierimel
  • Posts: 6
  • Joined: Nov 24, 2020
|
#82906
Could someone explain why B is the correct answer?
User avatar
 KelseyWoods
PowerScore Staff
  • PowerScore Staff
  • Posts: 1079
  • Joined: Jun 26, 2013
|
#82962
Hi kassierimel!

For this Local question, you need to do a mini-diagram with O on the 3rd day. If O is on the 3rd day, we know that M cannot be on day 2 (or day 7) or day 4. Since we already know that H cannot be on day 1 or day 5, this really restricts where that HM block can now go: HM can go 4/5 or 7/8. I would do 2 diagrams for this--one to show HM in 4/5 and the other with HM in 7/8.

If HM are in 4/5, then M would also have to be 1st. J would have to then be 2nd (so it is before H) which means it would also have to be 7th. That just leaves slots 6 and 8 for L and G. They are randoms so either one of them could go in either slot.

If HM are in 7/8, then H would also have to be 2nd. J would then have to be 1st (so it is before both Hs) which means it would also have to be 5th. That just leaves slots 4 and 6 for L and G. They are randoms so either one of them could go in either slot.

Those are the only possibilities if O is 3rd. Thus, answer choice (B) could be true--H can be 2nd if HM are in 7/8. None of the other answer choices are possible.
Screen Shot 2021-01-04 at 3.09.49 PM.png
Hope this helps!

Best,
Kelsey
 kassierimel
  • Posts: 6
  • Joined: Nov 24, 2020
|
#82971
Thank you!

Get the most out of your LSAT Prep Plus subscription.

Analyze and track your performance with our Testing and Analytics Package.